Calculus 8th Edition

Published by Cengage
ISBN 10: 1285740629
ISBN 13: 978-1-28574-062-1

Chapter 14 - Partial Derivatives - 14.5 The Chain Rule - 14.5 Exercises - Page 985: 38

Answer

$$8160 \pi \dfrac{in^3}{s}$$

Work Step by Step

We need to use chain rule as follows: $$\dfrac{dV}{dt}=(\dfrac{\partial V}{\partial r})(\dfrac{dr}{ dt})+(\dfrac{\partial V}{\partial h})(\dfrac{dh}{dt})$$ Now, we have: $\dfrac{\partial V}{ \partial r} \approx \dfrac{\partial }{ \partial r} (\dfrac{1}{3} \pi r^2 h)=\dfrac{2\pi r^2 h}{3}$ and $\dfrac{\partial V}{ \partial h} \approx \dfrac{\partial }{ \partial h} (\dfrac{1}{3} \pi r^2 h)=\dfrac{\pi r^2}{3} $ and $\dfrac{dV}{dt}=(11200 \pi)(1.8)+(4800 \pi)(-2.5)= 8160 \pi \dfrac{\ in^3}{s}$
Update this answer!

You can help us out by revising, improving and updating this answer.

Update this answer

After you claim an answer you’ll have 24 hours to send in a draft. An editor will review the submission and either publish your submission or provide feedback.